PT3.S4.Q05 : Corporate Opera Funding

reilly.j.doddreilly.j.dodd Core Member
edited December 2021 in Logical Reasoning 57 karma

Does anyone else feel like the correct answer on this one, C, attacks the premise directly? The Stem says that if we lower opera production costs so that they can be funded purely by box office tickets, we can do away with large corporate sponsorships which only play the most famous operas. AC C (for a weakener Q) states that operas cannot be funded without the help of large corporate sponsorships. Is this question one of those very rare ones where we do in fact attack the premise or am I misunderstanding something else about this question?

Comments

  • LivinLaVidaLSATLivinLaVidaLSAT Alum Member
    705 karma

    It sounds like you’ve mistaken the conclusion (last sentence of the stimulus) for a premise and you think answer C attacks the second half, which it does.

    Conclusion: If we reduce production costs so operas can be funded by tickets and individual donations, then public can see less famous operas.

    Premises:

    High production costs limit operas available (intermediate conclusion)

    If high production costs, then we need large corp sponsors

    Large corp sponsors only want most famous operas

    Chosen operas should be dictated by ticket purchasers

    Prephrase:

    Several assumptions came to mind but didn’t put effort into prephrasing because the conclusion is conditional. In this case, my primary strategy is to look for an answer that attacks the conditional conclusion.

    Simple rule for weakening a conditional conclusion:

    attack the necessary condition (NC) by showing that it doesn’t have to occur when the sufficient condition (SC) occurs.

    Answer C does exactly this. If no corp sponsors, then opera companies can only afford to produce most famous operas

    The SC here has the same meaning as the SC in the conclusion, but we get a different NC. Even without corp sponsors (meaning we fund costs with tickets and solo donations), opera companies still can only produce the most famous operas. This contradicts the conclusion.

    What would make someone not choose C?

    • Not familiar with simple rule for weakening a conditional conclusion
    • Can’t match up the language in the answer to the conclusion, so they don’t see the contradiction
    • The answer is the opposite of what you would expect. In the real world, you might assume the most famous operas to have higher production costs than less famous operas; however, the stimulus doesn’t give us this comparison. It’s missing and would be helpful in evaluating the argument.
  • reilly.j.doddreilly.j.dodd Core Member
    57 karma

    Thanks for this!

Sign In or Register to comment.